LSAT and Law School Admissions Forum

Get expert LSAT preparation and law school admissions advice from PowerScore Test Preparation.

 Administrator
PowerScore Staff
  • PowerScore Staff
  • Posts: 8917
  • Joined: Feb 02, 2011
|
#81022
Complete Question Explanation

Assumption. The correct answer choice is (D).

Answer choice (A):

Answer choice (B):

Answer choice (C):

Answer choice (D): This is the correct answer choice.

Answer choice (E):

This explanation is still in progress. Please post any questions below!
 allik@umich.edu
  • Posts: 5
  • Joined: May 03, 2016
|
#25117
Hello,

Can someone please explain #6 about the political analyst talking about the situation with McFarlane and Brooks. I narrowed it down to B and D but ultimately chose B because I didn't think it mattered if there were people that were indifferent, that would mean they weren't Brooks' supporters. Please explain this one to me!

Thank you!
 Robert Carroll
PowerScore Staff
  • PowerScore Staff
  • Posts: 1787
  • Joined: Dec 06, 2013
|
#25188
allik,

Answer choice (D) refers to supporters or opponents of MacFarlane. If the negation of that answer is true, there may be a substantial number of people indifferent to MacFarlane, as you said, but that doesn't make them indifferent to Brooks. Thus, they could be supporters of Brooks. Answer choice (D) then represents an assumption on which the argument depends.

There is no reason to assume that corruption has gone down. Thus, answer choice (B) is out of scope.

Robert Carroll
 mpoulson
  • Posts: 148
  • Joined: Mar 25, 2016
|
#25965
Hello,

I don't understand why answer is D and not C. I thought if the Mcfarlane's positions overlaped with Brooke's then she would most likely garner a more supporters and the argument in the stimulus is incorrect. Why specifically is C wrong? Thank you.

- Micah
 Ladan Soleimani
PowerScore Staff
  • PowerScore Staff
  • Posts: 43
  • Joined: Oct 08, 2015
|
#26117
Hey Micah,

The issue with answer (C) is that it doesn't directly address the reasoning that the author gives for why Brooks won't have many supporters. The author's argument is based on the fact that both supporters and opponents of McFarlane do not like Brooks. Supporters believe he is corrupt and opponents will oppose anyone who joins the government. You are looking for what is needed for this argument to be true. Since the reasons the author gives for why people don't support Brooks don't rely on his political positions, answer choice (C) is not required for the argument. Does that make sense?

Ladan
 mpoulson
  • Posts: 148
  • Joined: Mar 25, 2016
|
#30869
Yes, it does. Thank you.

Get the most out of your LSAT Prep Plus subscription.

Analyze and track your performance with our Testing and Analytics Package.